Вы находитесь на странице: 1из 68

MOCK TEST ONE – CLAT 2016 PATTERN

THE CLATGYAN TEST SERIES – 2016


IMPORTANT: The ownership of this test lies solely with CLATGyan; any
redistribution or reproduction of part or all of the contents in any form are
prohibited. You may not, except with our express written permission, distribute or
commercially exploit the content. Nor may you transmit it or store it in any other
website or other form of electronic retrieval system. Any kind of infringement of
these intellectual property rights would be taken seriously by CLATGyan and will
lead to criminal charges being pressed.

CLATGYAN 

SECTION 1: ENGLISH

Arrange the following sentences to form a coherent paragraph:


1. 1) As she started to walk past it, Michael Moretti stepped out. ―I‘ve been waiting for
you.‖
2) ―Hey,‖ he laughed, ―cool down. All I want to do is talk to you. All you have to do is
listen. I‘ll pay you for your time.‖
3) One afternoon as Jennifer was leaving the courthouse, she noticed a large, black,
chauffeured Cadillac limousine at the curb.
4) ―You‘ll never have enough money.‖
5) ―Get out of my way,‖ Jennifer said. Her face was flushed and angry, and she was
even more beautiful than he had remembered.
A) 34125 B) 31245 C) 31524 D) 32154

2. 1) His father, the king of Denmark, died suddenly and less than two months after this,
his mother married her husband‘s younger brother.
2) He was a dark and noble young man.
3) Hamlet was the Prince of Denmark
4) He suspected that his uncle had murdered his father.

CLATGyan | Batch of 2015, 2018, & 2019 – NALSAR University of Law Page 1
5) Hamlet mourned the death of his father.
A) 31245 B) 31524 C) 32145 D) 32154

3. 1) He gave him a good house to live in and several big farms to cultivate.
2) Nala returned to his kingdom with his wife and invited Pushkar to gamble with him
once again.
3) Nala became king once again, but he did not turn his unkind brother out of his
kingdom.
4) Pushkar lost the kingdom and everything he had.
A) 4312 B) 3421 C) 2431 D) 4321

4. 1) Once people get used to the Six Hats framework, they often change out of the
‗permanent caution‘ mode.
2) They come to welcome this opportunity to show their ability in more modes than
just one.
3) They perform well under the black hat- but now they also perform well under the
yellow hat and the green hat.
4) It is important to recognise the excellence and importance of a black hat and to
refrain from overuse of the black hat.
A) 4123 B) 1324 C) 1234 D) 4132

5. 1) On 4th July 2012, the ATLAS and CMS experiments at CERN‘S Large Hadron
Collider announced they had each observed a new particle in the mass region around
126 GeV,
2) On 8th October 2013, the Nobel Prize in Physics was awarded jointly to Francois
Englert and Peter Higgs ―for the theoretical discovery of a mechanism that contributes
to our understanding of the origin of mass of subatomic particles.‖
3) This particle is consistent with the Higgs boson predicted by the Standard Model.
4) Other types of Higgs bosons are predicted by other theories that go beyond the
Standard Model.

CLATGyan | Batch of 2015, 2018, & 2019 – NALSAR University of Law Page 2
5) The Higgs boson, as proposed within the Standard Model, is the simplest
manifestation of the Brout-Englert-Higgs mechanism.
A) 13542 B) 21543 C) 51234 D) 12534

Spot the error in the following sentences:


6. If you're saying (1) your vows in an airplane hanger (2), there's no better (3) focal point
than this (4).
A) 1 and 2
B) 1 and 3
C) Only 2
D) 2 and 4

7. Turning a vocation (1) into paying work can destroy your passion for it (2), so you
should plan ahead (3) and take it a step at a time (4).
A) 1
B) 2
C) 3
D) 4

8. The block system (1) permitted the Soviet Union (2) to exercise domestic control
indirectly (3) and led to excessive red tape bureaucratic measures (4).
A) 3
B) 3 and 4
C) 1
D) 1 and 2

9. Last night, on his way back from work (1), he was killed in a firey accident (2) which
took place after an inebriated truck driver (3) rammed into the divider on the road (4).
A) 1
B) 2
C) 3
D) 4

CLATGyan | Batch of 2015, 2018, & 2019 – NALSAR University of Law Page 3
10. Even after the numerous misunderstandings (1) and occasional tiffs, Rakesh could not
(2) help but maintain a (3) friendly raport with Neelam (4).
A) 1
B) 2
C) 3
D) 4

Given below are 3 passages. Choose the option that best answer the questions
that follow.

I. It is ironical that at a time when yoga is increasingly being recognised around the world as
an efficacious discipline that aids physical and mental well-being, the ancient Indian system
is caught in a needless controversy, mainly due to its aggressive promotion by the Narendra
Modi government. It is difficult to avoid the impression that the government is showing
excessive zeal as well as a tendency to use its employees and institutions to propagate its own
view of culture and tradition. Mobilising staff members and students seems to be this
regime‘s way of promoting an idea. If it was Good Governance Day last Christmas, it will be
International Yoga Day on June 21. It is indeed true that Prime Minister Modi‘s address to
the United Nations General Assembly in September 2014 provided the platform for the
international community to recognise the importance of yoga. In December, the UNGA
passed a resolution with the backing of over 170 countries to designate June 21 as
International Yoga Day. No doubt, the benefits of yoga ought to be widely disseminated.
However, does promoting it require the mobilisation of tens of thousands of people at
Rajpath in Delhi for a massive demonstration? There are apprehensions that employees and
students would be asked to participate in related events on a Sunday, even though it has not
been made mandatory.

The government is even aiming for an entry in the Guinness World Records for the single
largest yoga demonstration. It appears that having international impact is a key objective
behind the promotional activities. If yoga is all about health, peace and harmony, there really
is no need for a demonstrative approach to it. The visible presence of the state in the
promotion of yoga will only detract from the idea of making it a people‘s movement. Rather,
the government‘s role should be confined to providing facilities for the practice of yoga in
various institutions under it and disseminating information about its benefits. A related issue
that has given a sectarian dimension to the yoga campaign concerns a perception that the

CLATGyan | Batch of 2015, 2018, & 2019 – NALSAR University of Law Page 4
practice of yoga, especially the surya namaskar part of it, is against the tenets of Islam.
Recognising this, the government has dropped surya namaskar from the list of asanas to be
performed on June 21. While it is true that yoga is part of a wider heritage and attracts
practitioners from among adherents of various religions, the government is obviously unable
to convince everyone that its programmes are free of all religious or cultural association. It
should work to remove its initiatives from areas of contestation so that even programmes
having universal value do not take the hue of its ideology.

11. What is the synonym of the word ―efficacious‖ as used in the passage?
A) Incapable
B) Unproductive
C) Puissant
D) Enthusiastic

12. When and where did the Indian government stress on the importance of Yoga?
A) In June at Rajpath
B) In September in the General Assembly in the United Nations
C) In December in the General Assembly in the United Nations
D) In October in the Indian Parliament

13. According to the passage, what could be the nearest synonym to ―disseminate‖?
A) Promulgate
B) Collect
C) Gather
D) All of the above

14. What is the main flaw in the government‘s argument in favour of Yoga?
A) It is ignoring the wishes of certain sects in the country.
B) It is focussing only on the demonstrative aspect of the tradition.
C) It is unable to convince the people of the secular nature of Yoga.
D) Both B and C.

15. For what is Yoga being recognised around the world?


A) For focussing on the rich and sacred traditions.

CLATGyan | Batch of 2015, 2018, & 2019 – NALSAR University of Law Page 5
B) For aiding mental and physical well-being.
C) For its ability of bringing about peace and harmony.
D) For its universal value as it is accepted by major religions around the world.

16. In context of the passage, what would the word ―alacrity‖ mean?
A) Zeal
B) Dissemination
C) Efficacious
D) Regime

17. From a social perspective, what was the main argument against Yoga being propagated
as a tradition and culture?
A) It ignored the views of the population by catering only to the demands of the
Parliament.
B) Certain practices under Yoga were contrary to the religious beliefs of others.
C) There was no need for demonstrative approach to stress on the importance of Yoga.
D) The presence of state power will prevent it from becoming a people‘s movement.

18.What was the problem with June 21st being set as International Yoga Day?
A) It was a Sunday and hence would amount to people having to give up a holiday.
B) It went against the tenets of Islam since it was the day of Summer Solstice.
C) It would result in numerous cases of heatstroke since it was in peak summer.
D) Both A and C.

19. What were the aims of the government to observe an International Yoga Day, as given
in the passage?
A) It wanted to enter the Guinness book of world records.
B) It wanted to spread physical and mental well-being in the country.
C) It wanted to have an international impact, given the announcement being made at
the General Assembly in the United Nations.
D) All of the above.

20. What has the author of the passage suggested to the government as a remedy in future
such controversies?

CLATGyan | Batch of 2015, 2018, & 2019 – NALSAR University of Law Page 6
A) Remove parts of the initiative that offend other religious denominations
B) Students should be briefed on the importance of the initiative in their classrooms.
C) Remove initiatives that may take an ideological hue despite them having a
universal value.
D) Both A and C.

II. Don‘t get fooled by her homely Kanjeevaram clad looks. This feisty female is determined
to dispel the clichéd notions the external world has about India. And Lakshmi Pratury
believes that the change in perception has to begin at home. ―We need to look at the new
India with new eyes and focus on bringing digital access to everyone and cater to the needs of
the youth in a whole new way. We need to give them a platform to shine and we need to listen
to them more,‖ says the charming lady.

Lakshmi, has been attending TED since 1993, decided to bring it to India in 2009. She
compares this feeling to one of bringing your best friend home. ―The independence that Chris
Anderson gave me to co-curate, identify the stories and interact with India opened my eyes to
the stories that exist here and need to be told. It also showed the power of the stories –
Pranav Mistry‘s talk is still very popular. Talks by people like Devdutt Patnaik and Sunitha
Krishnan catapulted them from being local intellectuals to being global personalities,‖ she
recollects.

When asked by actor/director Farhan Akhtar to talk about what drives her, Lakshmi admitted
her love for theatre fuelled the creation of a single stage which would connect innovators and
knowledge-seekers from all around the world.

In December 2010, Lakshmi curated and hosted the first INK Conference, in association with
TED. This was a massive large-scale effort and she attributes its success to two factors. ―You
need an audience who values a great experience and you require corporations who see the
value of being associated with the conference. Initially, it seems tough but in the end, it was
great collaboration with TED to bring close to 1,000 people from over 40 countries to India,‖
she states.

INK has always been on the lookout for out-of-box thinkers who are creating an impact in
this world. Some of the speakers at the INK Conference include Bhavesh Bhatia – visually
challenged entrepreneur, Arunima Sinha – mountain climber, Rajkumari Hirani – filmmaker
among others. ―To date, we posted over 250 videos on inktalks.com as well as other platforms

CLATGyan | Batch of 2015, 2018, & 2019 – NALSAR University of Law Page 7
with over 17 million views. It is truly gratifying when I travel across India and have young
people tell me how much they love the videos that we post – that they are proud to have new
role models,‖ she reveals.

There is a restless soul that exists within this multifaceted personality who has straddled
various mediums be it theatre, literature, creative writing, marathon running, rock climbing,
snow climbing, river rafting and a whole other range of habits. ―Each activity taught me how
to be with different set of people, learn the rules of the game, know when to push, when to
hold back and taught many valuable life lessons. I get bored of things easily. Also, I do not
have the need to be the best in any one area. I love a little bit of a lot of things so that I can
understand what motivates people in that area. So every year, I used to take on a different
activity to keep myself engaged and learn forever,‖ she adds.

21. What does Lakshmi Patrury think will dispel the clichéd notions of India?
A) Providing people with access to technology.
B) Increasing job opportunities for the middle-aged.
C) By allowing corporations to set up their offices in the metro cities in India.
D) By procuring the right kind of recognition.

22. Who is the ―multifaceted personality‖ being referred to in the passage?


A) Arunima Sinha
B) The youth of India
C) Lakshmi Patrury
D) Rajkumari Hirani

23. Which incident did Lakshmi Patrury think was similar to that of bringing your best
friend home?
A) Chris Anderson giving her the freedom to identify stories and interact with India.
B) Listening to talks by renowned intellectuals and feeling an inner connection.
C) Organising the first INK Conference by gathering large-scale support.
D) Introducing TED to India.

24. What would Lakshmi Patrury not consider ideal for the success of a conference?
A) An audience that gives importance to a great experience.

CLATGyan | Batch of 2015, 2018, & 2019 – NALSAR University of Law Page 8
B) Corporations that would want to be associated with the conference.
C) Posting videos online for people to find their role models.
D) Both B and C

25. Which of the following statements is not true about Lakshmi Patrury as given in the
passage?
A) She gets bored of things easily.
B) She is most passionate about theatre which was instrumental in her work.
C) She brought TED to India in 2009.
D) She wanted to be on the top in whatever she decided to do.

III. ―My dream is definitely to become a Formula One world champion, if I can do it more
than once that would be even better,‖ states Arjun Maini. At an age when his peers enjoy the
simple pleasures and pressures in life – be it hanging with their friends, reluctantly waiting
for their results or pushing the boundaries in an attempt to figure out their life‘s true interests
as they slowly but surely march towards inevitable adulthood – Maini is a rarity.

The youngster from Bengaluru has already sacrificed the pleasures of a teenage to pursue his
dream. Life on the road for a young driver can be arduous with one too many lonely nights to
go along with the physical and mental demands of the sport.

And as a teenager and student, education is yet another ball to juggle. ―It does get quite hectic
but my school has been extremely supportive. They email my parents and tutor the
curriculum and have been flexible in terms of my attendance and exam schedule,‖ reveals
Maini.

―I have a few hobbies and a PlayStation to keep me occupied during the days I want to relax. I
get along fairly well with my teammates along with members from my team so the thought of
being lonely doesn‘t really cross my mind. We all have a different view of what fun is and for
me I‘m having the time of my life driving a Formula 3 car at its limit,‖ says the 17-year-old
with impressive maturity.

One thing that helps him is the presence of family and their unsullied support in his
endeavour. ―Family support has probably been the most important thing for me so far. More
than even the funding, a family that supports you is almost integral to succeed in this sport,‖
stressed the Bengaluru lad, whose brother Kush is also making ripples in motorsports.

CLATGyan | Batch of 2015, 2018, & 2019 – NALSAR University of Law Page 9
One of the hardest things in life is to find one‘s calling and hold fast amongst a multitude of
distractions while working steadfastly towards achieving the goal. Maini is among those rare
few who are blessed with the aim, the drive and talent – the whole package.

Reason enough, one might argue, for the teen to be hailed as the poster boy of Indian
motorsports. Blazing a trail of success at different levels in his wake, Maini is inching closer to
fulfilling his dream as he currently stands 17th in the driver‘s standings after six rounds in
Formula 3.

For any budding driver, Formula 3 is the crucial and big step towards the glitzy and
glamorous lights of Formula One. The recent jump of Max Verstappen, who raced for Maini‘s
team Van Amersfoort Racing in F3 last year to Formula One to become the youngest driver
ever in the championship only adds weight to the importance of the F3 circuit. But Maini
would rather keep his head down and pug away rather than get carried away with the magical
world that seems ever so close.

With Indians slowly making their presence felt as speed demons in a sport that is
predominantly dominated by European countries, Maini believes the reputation of Indians in
his circle of work is definitely on the rise. ―The reputation of Indian drivers is definitely
improving. Narain and Karun driving in Formula One has definitely changed a lot of things
for Indians looking for a career in motorsport,‖ he revealed.

If his past is any sign for his future, there is little doubt that Maini, who considers his first in
in the BRDC F4 car as his most amazing moment so far, is on an expressway to success.

26. What can be concluded from the passage given above?


A) Maini cannot risk his dream by pursuing the activities that his age group usually
engages in.
B) Indians have always been reputed in the motorsport circuit due to their success at
various events.
C) Formula 3 is the next big step from Formula One.
D) Maini feels that his team is united on the aspect of having fun and due to this, he is
never lonely.

27. What is the opposite of ―unsullied‖ as used in this passage?


A) discouraging

CLATGyan | Batch of 2015, 2018, & 2019 – NALSAR University of Law Page 10
B) tarnished
C) dissuasive
D) persuade

28. Which of the following statements is false with regard to Maini‘s background as given
in the passage?
A) Maini‘s life isn‘t as lonely as it is assumed to be since he shares a good rapport with
his teammates.
B) Maini‘s parents have been very supportive and he gives this a greater leverage than
being provided with funds.
C) Max Verstappen‘s record of being the youngest driver in Formula One does not
pressurize Maini to enter the Formula One circuit.
D) Maini‘s elder brother is his inspiration having won many laurels in motorsports.

29. Which trait of Maini‘s personality can be glimpsed at in this passage?


A) Maini is a fun loving person and always keeps himself occupied.
B) Maini is academically bright and is always up-to-date with his school work.
C) Maini is a family person and places support from family at the highest pedestal.
D) Maini is ambitious and seeks to break the record of Max Verstappen.

30. Which of the following words can replace the word ―arduous‖ that has been used in the
passage?
A) Vexing
B) Herculean
C) Disturbing
D) Rankling

IV. Firstly, congratulations to the Parsis of Kolkata, for once again demonstrating the
civilised response of our great and good community. I note that your response to American
hip-hopper Snoop Doggy Dogg‘s video displaying the ―Fravahar‖ has not been a fatwa of
death or an invasion of the record company‘s offices armed with Kalashnikovs. You have
taken the path of moderation and applied to an Indian court to ban the video.

My sincere advice would be to withdraw the case. It has and will increase the international
viewership of Snoopy‘s video, just as the attempt to ban Lady Chatterley‘s Lover projected the

CLATGyan | Batch of 2015, 2018, & 2019 – NALSAR University of Law Page 11
book as an all-time bestseller and the fatwa sold millions of Salman Rushdie‘s The Satanic
Verses.

I confess that I make an effort to acquire banned stuff out of natural curiosity and perverse
instincts. Besides, videos are free and a Kolkata court will, like our Achaemenid Emperor
Xerxes, who (according to the lying Greek historians) ordered the unruly waves of the
Hellespont to be whipped, fail in its attempt to suppress it.

I further confess to not sharing the Kolkata indignation. I disagree with Kolkata for several
reasons — theological, historical and cultural.

The figure that Dogg has made famous in the ―offending‖ video is what we Zoroastrians call
the ―Fravahar‖. The figure first appears in Bisotun on an inscription of Emperor Darius who
mentions his devotion to Ahura Mazda several times. It has been assumed by many
historians, including the great R.Z. Zaehner, that this human figure in the circle with wings is
a depiction of Ahura Mazda. If that is so, then using an image of God to decorate the set of a
video that isn‘t strictly devoted to matters eschatological is, at the least, bad taste, and at
worst blasphemy. Blasphemy is punished as a sin. Joan of Arc was, so to speak, fired. The
Zoroastrian preachers Mani and Mazdak were declared blasphemers by the Dasturs of
Sasanian times and executed. The Sunni fanatics of Islamic State of Iraq and the Levant
(ISIL) are even today beheading and crucifying people for what they denounce as blasphemy.
Zoroastrians stopped the chopping some centuries ago, leaving it to God to strike the guilty
down with bolts of lightning, with slippery banana skins or other divine weapons.

However, Herodotus, the father of history, very clearly says that the Achaemenids, the
Persian Zoroastrians of his time, unlike the Greeks, did not anthropomorphise their God. In
other words, Ahura Mazda was not as were Zeus, etc., ever portrayed in human form.
Historians, today, don‘t believe the figure in the winged chariot is Ahura Mazda. He is a spirit,
an angel or someone who intercedes between humans and the unfathomable divine.

31. What can be assumed from the passage given above?


A) Owing to the ban, more people will be curious to watch Snoop Dogg‘s video.
B) The author of the passage is a Parsi.
C) The author has a tendency to procure objects that our banned to feed his curiosity.
D) The author considers the path taken by Parsis of Kolkata as one of moderation.

CLATGyan | Batch of 2015, 2018, & 2019 – NALSAR University of Law Page 12
32. What could possibly be the reason behind the author citing examples of Joan of Arc
and Sunni fanatics in the passage?
A) To show how intolerant the religious heads were in the bygone era.
B) To realise the true nature of religious symbols.
C) To portray what the consequences of blasphemy could be.
D) Both A and C.

33. What has Snoop Dogg done that has offended the Parsis of Kolkata?
A) He has made fun of the incapability of the Achaemenid Emperor to whip the unruly
waves of Hellespont.
B) He has ridiculed Ahura Mazda by anthropomorphising him.
C) He has blasphemed by portraying himself in a winged chariot.
D) He has used the figure called ―Fravahar‖, which is considered to be a depiction of
Ahura Mazda.

34. What has been mentioned about Herodotus in the passage given above?
A) He believed that Parsis of his time did not try to give human shape to their Gods.
B) He didn‘t believe that the winged chariot is Ahura Mazda.
C) He encouraged the idea of leaving the punishment to God.
D) He thought that the Greek historians were lying about their opinion of Xerxes.

35. What does ―fatwa‖ mean as used in this passage?


A) A ban on any activity that is considered to be opposed to moral life.
B) A general opinion as to how one should govern oneself.
C) A ruling on a point of Islamic law given by a recognised authority.
D) A proposal to reject a decision made earlier.

Choose the most appropriate word that fits to make a coherent paragraph:
Prime Minister Narendra Modi must be (36) for his brave move in announcing that he will
visit Pakistan. I do not mean brave (37) the point of view of physical courage. It is clear that
Mr Modi will find that he is given security of the highest standard. But even so, even
Pakistan‘s most (38) man, President Musharraf had his convoy bombed twice and its formed
PM Benazir was killed. So Mr Modi is brave in agreeing to go.

CLATGyan | Batch of 2015, 2018, & 2019 – NALSAR University of Law Page 13
The second way in which he has been brave is that he has challenged many in our media and
also our strategic affairs experts in reaching out to Pakistan decisively. More importantly he is
(39) BJP supporters who insist that Pakistan be dealt with firmly or not at all. Mr Modi has
for long time been celebrated for thumbing his nose at Nawaz Sharif. India for the last one
year has said it will be able to (40) Pakistan to its terms. This was the reason that India sulked
with Pakistan over non-issues like the Hurriyat meeting the Pakistani High Commissioner.

36. A) flattered
B) congratulated
C) criticised
D) satisfied

37. A) through
B) for
C) on
D) from

38. A) reputed
B) famous
C) protected
D) misunderstood

39. A) defying
B) escaping
C) obeying
D) believing

40. A) curve
B) bend
C) twist
D) buckle

CLATGyan | Batch of 2015, 2018, & 2019 – NALSAR University of Law Page 14
SECTION 2: LOGICAL REASONING

The problem for the CPI(M), however, remains to be that it has still not managed to offer
itself as an agent of change in the State, and is perceived warily by an electorate that had
seen three and a half decades of Left rule. Despite a change in its State leadership, the
CPI(M) still has little to offer as an oppositional force. Meanwhile, the Trinamool
Congress, by virtue of its posturing toward the more populist side of the Left, has earned
the trust of the rural poor to a substantial degree. While a new alliance strategy, including
an engagement with the Congress, could help the Left arithmetically, the inability to
resuscitate its fortunes through internal reorganisation, and the absence of any movement
towards pan-Left unity must be cause for greater worry for the CPI(M). The Left‘s state of
crisis is not limited to West Bengal. In Kerala, recent by-election results indicate that the
Congress-led United Democratic Front is not facing any imminent threat on account of
anti-incumbency factors, while the CPI(M) faces issues of factionalism. In Tripura it is
holding on its own, although challenges are evident. The CPI(M) and other Left groups
have been harping on the slogan of ―Left unity‖ and a joint struggle, but this has remained
more of a goal on paper than one that could be translated into anything meaningful on the
ground. The CPI and the CPI(M), for example, seem to have little difference in terms of
essential ideology anymore, but even in their most weakened and dire straits, such as now,
they have not really pushed the envelope on a unification initiative. The future, thus,
admittedly remains foggy.

41. What is the inherent assumption being made in this passage?


A) That the electorate is wary because it has seen three and a half decades of Left Rule.
B) The Trinamool Congress has earned the trust of the rural poor by posturing
towards the more populist side of the Left.
C) The Left‘s state of crisis extends beyond West Bengal.
D) Issues of factionalism are a cause for the Left‘s state of crisis.

42. Which of the following statements, if true, most weakens the author‘s conclusion in the
above passage?
A) The ideologies of the CPI and the CPI(M) have always been the same.

CLATGyan | Batch of 2015, 2018, & 2019 – NALSAR University of Law Page 15
B) The goals that the CPI and the CPI(M) seek to achieve are the same.
C) Pushing the envelope on a unification initiative will not help in solving the crisis of
the Left.
D) The crisis of the Left is not so much due to ideological differences and the need for
a unified movement, but rather due to inefficient bureaucratic decisions.

43. Which of the following is not an inference that can be drawn from the passage?
A) The CPI(M) plays a role as an opposition party.
B) The Left‘s crisis extends to other states outside of West Bengal.
C) Historically, the CPI and the CPI(M) have operated on different ideologies.
D) In Kerala, the United Democratic Front faces issues of factionalism.

44. What is the assumption being made in the first sentence of the passage?
A) The electorate being referred to is that of West Bengal.
B) The CPI(M) aims to offer itself as an agent of change in the state.
C) The electorate is wary off Left Rule.
D) There is no assumption underlying the first sentence.

45. Strangely in Nalsarland, there are several small stores that sell food at prices far higher
than elsewhere in Hyderabad city. After all, there are many supermarkets in the city
that sell food at cheaper prices, and many of these supermarkets are open 24-hours.

Which of the following, if true, would be least helpful in explaining the paradox stated
above?

A) These stores sell special foods that are not available in super markets.
B) The stores provide home delivery.
C) These stores mainly sell stationery; food is only a small part of their business.
D) These stores have been around for generations, and are an integral part of the
community, whereas the supermarkets are new and unfamiliar to the locals.

Bureaucrats have social status and power. They also possess personal knowledge of the rules.
So, as a reasonably educated Indian citizen knows, it is pointless to argue with a bureaucrat:
one can only plead with him or her. The case of politicians is different. They seem to be
one of “us”, as they derive their power from their popularity among ordinary

CLATGyan | Batch of 2015, 2018, & 2019 – NALSAR University of Law Page 16
men and women. Also, we learn at school that democratic politics is all about discussion
and debate. So, when we grow up and attain adult citizenship, we live in the hope that our
views will be heard by those who represent us. This hope — and the belief that keeps it alive —
received a body blow during the 21 months of Emergency rule, in effect from June 25, 1975
until its withdrawal on March 21, 1977. Ordinary citizens realised with shock that they had no
rights whatsoever under the new system. When Indira Gandhi became supreme leader, the
politicians surrounding her became small dictators. The new style of exercising
control was copied all the way to the municipal level. Civil servants radiated jubilation over
the clarity of orders they had from above. Many prominent public voices turned into
sycophants.

46. How are the two sentences highlighted in bold related to each other?
A) The first is a description of politicians, whereas the second is the author‘s
observations of the effects of Emergency on politicians.
B) The first is the author‘s proposition, whereas the second is his conclusion.
C) The first is an assumption underlying the second.
D) The first and the second are unrelated sentences.

47. Which of the following is an inference that can be drawn from the article?
A) Democratic politics involves discussions and debates.
B) Citizens hope that their views are heard by their representatives.
C) Prior to the Emergency, civil servants did not receive clear orders.
D) Ordinary citizens live under a dream of hopefulness.

48. Which of the following most weakens the argument that people grow up to believe that
their views will be heard?
A) Learnings in school do not really have an impact on shaping one‘s belief as an
adult.
B) One internalises ideas that he has grown up learning.
C) Beliefs in adult life are drawn from past experiences and learnings.
D) None of the above.

49. Too many women depend on men to make them happy, and I would be lying if I said I
haven't done that myself. At this point in my life, I don't know how to be completely

CLATGyan | Batch of 2015, 2018, & 2019 – NALSAR University of Law Page 17
independent when I'm in a relationship. I know many girls who can be, but personally,
I don't know how.
What is the underlying assumption that the author makes?

A) Women cannot be happy without depending on men.


B) The author herself has depended on men for happiness.
C) The author wants to learn to be independent.
D) Relationships cause one to be dependent on their partners.

50. Manav: Airplanes contribute more carbon dioxide to the atmosphere in one year than
does the whole of Africa. If we want to reduce global warming we need to restrict air
travel.

Ajey: Did you know that by taking one inter-continental flight, you cause more
pollution than you would in twelve months of car travel?

Ajey‘s response to Manav‘s comment serves to

A) Reinforce Manav‘s argument that airplanes contribute hugely to increased levels


of carbon dioxide.
B) Weaken Manav‘s approach in his argument
C) Add more weight to the contention that we should reduce air travel
D) Question whether Manav really understands global warming

51. The committee on sexual discrimination in the workplace has highlighted Manliness
Ltd. as a chief offender. Of all the ten senior executives in the firm, only one is a
woman. And of the twenty junior executives, only five are female.
Manliness Ltd. could best defend itself against the charges by showing that:
A) Male and female executives at the same level have the same qualifications
B) The salary paid to men and women at equal positions is the same
C) Five times more men than women apply for jobs with the company
D) The best job applicants were accepted

52. Kshitij has twenty years of playback singing experience behind him; therefore, if you
are looking for a good singer for your latest album, look no further.

CLATGyan | Batch of 2015, 2018, & 2019 – NALSAR University of Law Page 18
The speaker assumes that:

A) Twenty years of practice ensures good singing


B) The kind of singing required for the album is the same as that practiced by Kshitij
C) Kshitij is the best singer who can be found
D) Kshitij will do a decent job

Verbal critical reasoning tests are often described by job seekers as the most difficult type of
aptitude tests. There is very good reason for this. In most psychometric tests you have to find
or use key information in the text to answer the questions. For example, in many verbal
reasoning tests you are presented with a statement on the passage and asked to decide
whether the statement is true, false or cannot say. This means you are simply asked to find
the statement in the text. Basically, these are fact finding missions. This generally isn‘t the
case in verbal critical reasoning tests. In critical reasoning tests you only use the text as the
basis to your understanding and not to actually answer the question.

53. Which of the following is an assumption that the author makes?


A) Verbal critical reasoning requires more skill than psychometric tests.
B) Understanding the text is more difficult than finding a sentence in the text.
C) It is easy to ascertain if a statement is true or false.
D) Verbal critical reasoning is a better judge of aptitude.

54. ―The article seems to think that world over, women talk the way American
women talk, and the way American women talk is apparently the most genuine and...‖
Which of the following is the most logical completion of the sentence above?
A) the best at connecting with other people.
B) pleasing way.
C) well articulated.
D) well thought out.

55. Indian cuisine is highly regarded all over the world for its healthy ingredients
and unusual mix of flavours. Yet in India, there are more fast food restaurants selling
burgers and fries (which many people now class as ‗junk food‘) than there are in any
other country. Indians evidently love junk food.

CLATGyan | Batch of 2015, 2018, & 2019 – NALSAR University of Law Page 19
Which of the following, if true, would most weaken the author‘s contention?

A) There are also a large number of Vietnamese restaurants in India than in other
countries.
B) Traditional Indian cuisine is very expensive in restaurants.
C) There are an unusually large number of tourists in India who prefer eating at fast
food joints.
D) Junk food actually has higher nutritional value and can sustain hunger for a longer
period of time.

56. Scientists investigating a rare metabolic disorder hypothesized that hairfall was an
important symptom in the development of the disease. A study of twenty patients found
that, on average, the patients had close to the normal amount of hair for their age.

Before concluding that hairfall was not an important symptom, the researchers would
find the answer to which of the following questions, NOT useful?

A) Is the amount of hair dependent on age?


B) How many of the patients had hairfall that was genetic?
C) Did the patients have any other diseases that caused hairfall?
D) Were the patients also obese?

57.Deka: We have too many people working on the network. The high frequency of
breakdowns is due to too much traffic on the network.

Anna: We have just as many people using the network at my college, yet we hardly
need any repairs to our systems. Our systems must be better than yours.

Anna‘s argument would be most strengthened by providing data on the

A) Actual number of people using the networks in both colleges


B) The type of network systems used
C) Ratio of networks to users
D) Number of networks in Deka‘s college

CLATGyan | Batch of 2015, 2018, & 2019 – NALSAR University of Law Page 20
58. A rare disease caused by the tse tse fly is being diagnosed with increasing frequency.
The number of cases reported this year is more than double the number reported four
years ago. The government should now allocate more funds for treatment and
prevention of the disease.

All of the following, if true, would weaken the conclusion except

A) Funds already available are under utilised.


B) A new test has been discovered this year, that detects the disease at the earliest
stage.
C) A private organisation has committed sufficient funds for research on the
disease.
D) None of the above.

59. Question: Should the government increase efforts towards the protection of
endangered species?
Argument I: No, one can never ascertain what is meant by the term ‗endangered‘.
Argument II: No, species extinction is inevitable. Why bother, and waste funds?

A) Only argument I is strong


B) Only argument II is strong
C) Both arguments are strong
D) Neither argument is strong

60. Either Asad or Aymen will leave for Delhi if it is Monday tomorrow.
Which of the following logically follows from the above statement?

A) Asad will leave, so it is Monday tomorrow.


B) Either Asad or Aymen will leave, so it is Monday tomorrow.
C) It is not Monday tomorrow. Neither Asad nor Aymen will leave.
D) Neither Asad nor Aymen will leave. Therefore, it is not Monday tomorrow.

Answer the following questions based upon the information provided:


a) Six persons A, B, C, D, E and F live in Nelamangala Township.

CLATGyan | Batch of 2015, 2018, & 2019 – NALSAR University of Law Page 21
b) In the group, there are two married couples. No male member in the group is a Clerk
or an Actor
c) The Clerk earns less than her husband, who in turn earns the least among the Lawyers.
d) Among the three Lawyers, husband of D, with whom his old aged father lives, earns
more than E, who in turn gets more pay than B.
e) F (old age man) lives with his married son C.
f) The wife of one of the Lawyers is an Actor.
g) The Actor earns least in the group.
h) Out of six persons, three are Lawyers, one Clerk and one Actor.

61. Who gets the highest salary?


A) A
B) B
C) C
D) D

62. Which of the following is a married couple?


A) AB
B) CD
C) EB
D) The answers are not clear.

63. Which of the following is a pair of female members in the group?


A) BC
B) CD
C) AB
D) AD

64. Who is the actor in the group?


A) C
B) D
C) E
D) Insufficient Information

CLATGyan | Batch of 2015, 2018, & 2019 – NALSAR University of Law Page 22
65. Who is the husband of the actor?
A) A
B) B
C) C
D) D

66. ‘X‘ started walking straight towards North. He walked a distance of 5m and then took
a right turn and walked a distance of 3m.Then he took a left turn and walked a distance
of 5m again. ‘X‘ is facing which direction, now?
A) North
B) South-East
C) West
D) North-West

67. Ram was facing West from where he turned to his right and walked 12 ft., then he
turned towards left and walked 6 ft., After that he walked 6 ft. in North direction and at
last I walked 6 ft. in the East. Then, in which direction I am standing from the original
point?
A) North
B) South-West
C) South
D) North-West

68. Ananya started from a point ‗A‘ towards North and travelled 5 km. Then she turned
left and travelled 2 Km. Then she turned left and travelled 5 km. Then she turned right
and travelled 5 km. How far is she from the point ‗A‘?
A) 5 km
B) 7 km
C) 9 km
D) 11 km

69. Akbar starts from a point A and walks 5 m towards South-West direction and reaches
point B. From here he travels 8 m in West direction and reaches point C. From C he
travels towards North-East direction and reaches point D after traveling a distance

CLATGyan | Batch of 2015, 2018, & 2019 – NALSAR University of Law Page 23
equal to AB. At last, he turns towards East direction and reaches point A. How much
distance has been covered by Amit?
A) 26m
B) 24m
C) 22m
D) 28m

70. Tanvi travels 2 m in South direction; she turns left and travels 4 m. Again she turns
towards left and travels 5 m, then she turns towards right and travels 3 m, in which
direction and which direction is Tanvi from her original position?
A) North-East
B) North-West
C) South-East
D)South-West

Answer the Q. 71 and Q. 72 based on the information provided.

There are 6 members in a family, Aarant, Anand, Angeline, Aninidita, Anurag, Amritha.
There are two married couples, Anindita is grandmother of Aarant and mother of Anand,
Angeline is the wife of Anand and mother of Amritam, Amritha is the granddaughter of
Anurag.

71. Who among the following is one of the couples?


A) Aarant and Amritha
B) Anand and Anurag
C) Anand and Angeline
D) None of the above.

72. How Aarant and Amritha related?


A) Siblings
B) A Couple
C) Grandfather- Granddaughter
D) Nephew and Uncle

CLATGyan | Batch of 2015, 2018, & 2019 – NALSAR University of Law Page 24
73. A woman said to a man, your mother‘s husband‘s sister is my aunt. How is that man
related to that woman?
A) Brother
B) Father
C) Uncle
D) None of the Above

74. Rani introducing Ram says, His father‘s only son is my father. How is Rani related to
that Ram?
A) Aunt
B) Mother
C) Daughter
D) None of the Above

75. Pointing to a man in a photograph a woman says ―He is the son-in-law of the father of
my mother‘s only granddaughter‖. How is he woman related to the man?-
A) Mother
B) Aunt
C) Daughter
D) None of the Above

76. 7, 24, 25, 11, 60, 61, 13, 84, ?


A) 85
B) 86
C) 87
D) 88
E)
77. 1, 8, 9, 64, 25, 216, 49, 512,
A) 91
B) 117
C) 81
D) 77

78. ACE, GIK, MOQ, SUW

CLATGyan | Batch of 2015, 2018, & 2019 – NALSAR University of Law Page 25
A) SUV
B) SUW
C) UWY
D) UVY

79. If A=26, B=25… Y=2, Z=1,


What is the value of the word TUMATI PURNA SRI HARSHA?
A) 283
B) 281
C) 287
D) 285

80. 1, 5/4, 3/2, 7/4, ?


A) 2
B) 4
C) 7/6
D) 11/4

CLATGyan | Batch of 2015, 2018, & 2019 – NALSAR University of Law Page 26
SECTION 3: MATH

81. Which of the following options could be 18X + 21Y, where X and Y are natural
numbers?
A) 200
B) 201
C) 202
D) 203

82. If LCM of two numbers is 123, and the product of two numbers is 1599, the HCF
would be
A) 13
B) 17
C) 19
D) 11

83. 6!=
A) 750
B) 702
C) 720
D) 702

84. The average weight of a group of 30 people is 27. If one is removed, the weight of the
group decreases to 24. So what is the weight of the person who has been removed?
A) 114
B) 113
C) 112
D) 101

85. tan45 =
A) 1/2
B) 3/2
C) 1
D) 0

CLATGyan | Batch of 2015, 2018 & 2019 – NALSAR University of Law Page 27
86. Fanta borrows 2000 from Pepsi at 15% rate of interest which is compounded
annually. How much does Fanta have to pay after the 2nd year? (Use depreciation
funda)
A) 2645
B) 2650
C) 2675
D) 2680

87. Difference between two numbers is 1, and sum of the squares is 265. The numbers
are?
A) 9 and 10
B) 12 and 13
C) 11 and 12
D) 15 and 17

88. The probability for queen‘s heart to occur in a game of cards is?
A) 1/4
B) 1/52
C) 1/13
D) None of the Above

89. Find the odd one out:


A) 69, 260, 269
B) 115, 252, 277
C) 161, 240, 287
D) 160, 231, 281

90. When asked by the teacher to multiply x with 21, Chintu multiplied with 31 and the
resultant sum was 70 more than what he would have got if had multiplied with 21. So
what is x?
A) 9
B) 7
C) 11
D) 8

CLATGyan | Batch of 2015, 2018 & 2019 – NALSAR University of Law Page 28
91. Three fourth of a tank is full with water. If 5 litres are added to it, then four- fifth of the
tank becomes full. What is the capacity of the tank?
A) 75 litres
B) 80 litres
C) 100 litres
D) 120 litres

92. A cistern can be filled by two pipes in 20 minutes and 30 minutes respectively. There
is an outlet which empties the vessel in 16 minutes. If all are how long with it take to fill
the cistern?
A) 48 minutes
B) 60 minutes
C) 50 minutes
D) 54 minutes

93. A man takes 5 hours to walk a certain place and return by car. He would have gained 3
hours if he travelled both ways by car. How long would he have taken to walk both
ways?
A) 12 hours
B) 8 hours
C) 4 hours
D) 9 hours

94. If a man takes 3 hours to row 3 km upstream or 15 km downstream, then the speed of
the current in kmph is
A) 4
B) 9
C) 2
D) 6

95. A, B and C can do a piece of work in 6, 12 and 24 days respectively. They all together
will complete the work in
A) 5 2/7 days
B) 4 3/7 days

CLATGyan | Batch of 2015, 2018 & 2019 – NALSAR University of Law Page 29
C) 3 3/7 days
D) None of these

96. A and B can do a piece of work in 18 days, B and C in 24 days, C and A in 36 days. A
alone can do the work in?
A) 48 days
B) 56 days
C) 40 days
D) None of the above

97. An investor in shares makes a profit of Rs. 920 in his fifth investment, by increasing
his average of profit of first four investments by Rs. 14. His average profit over the first
four investments is
A) 906
B) 976
C) 850
D) None of the above

98. Raju wants to know the angle between the hour and the minute hand when his college
breaks for lunch which is 1:15?
A) 150
B) 60
C) 115
D) 120
99. The members of three main political parties in the country were in the ratio of 1:2:3.
This year, there has been an increase of members to the extent of 25%, 50%, and 75%.
So what is the current ratio?
A) 5:13:21
B) 7:12:23
C) 9:13:21
D) 5:12:21

100.Rs. 680 is divided among A, B, C such that A gets 2/3rd of what B gets and B gets 1/4th
of what C gets. A‘s share is
A. 75

CLATGyan | Batch of 2015, 2018 & 2019 – NALSAR University of Law Page 30
B. 80
C. 90
D. 100

CLATGyan | Batch of 2015, 2018 & 2019 – NALSAR University of Law Page 31
SECTION 4: LEGAL APTITUDE

101. Principle – Violation of a legal right, with or without actual damage, gives rise to a
tort.
Facts – Ravi and Lakshman are not the best of friends. They have strained relations
and they constantly compete against one another for better business prospects. Ravi
sets up a grocery store where he sells food grains for Rs. 20 per kilogram, a rate that is
substantially lower than other stores in the same area. The store is a huge success, and
Ravi gains huge profits, as everybody prefers to buy the grains from his store. Soon
after, Lakshman sets up another store next to Ravi‘s and puts up a board that says,
‗Rice – Rs.15 per kilo & Wheat – Rs. 12 per kilo‘. Attracted by the new offer, all of
Ravi‘s customers now prefer to buy grains from Lakshman‘s store. Ravi, hence, incurs
heavy losses and is forced to close down his store. He believes that Lakshman has set
up the shop in order to make his business fail, and decides to sue Lakshman for
damages. Will Ravi succeed?
A) No, Ravi cannot sue Lakshman, as Lakshman had an equal right to set up a store
and sell commodities at his own prices.
B) No, Ravi will not succeed, as no legal right has been violated by Lakshman‘s act of
setting up a store and selling food grains for a cheaper price.
C) Yes, Ravi will succeed, as he has suffered great losses, and the damage he has
suffered must be compensated by Lakshman.
D) Ravi can sue, as Lakshman has committed a tort.

102. Principle 1- When two parties enter into a contract and there is a breach of
contractual terms, only the party to the contract affected by such breach may sue.
Principle 2 – In the case of tortious liability, a duty of care is owed to the world at
large, and anybody affected by such breach of duty of care can sue for damages.
Facts – Ramesh, an ophthalmologist, has a 19 year-old son named Aditya. Aditya,
during a game of football, sustains a severe leg injury that needs to be surgically set
right. Ramesh enters into a contract with his friend Karthik, an orthopedic surgeon to
operate upon his son and cure his injury. However, as Karthik is operating, he leaves
behind a small cotton swab in the body of Aditya, and as a result, Aditya has to
undergo severe pain and suffering as well as a second surgery.
In this case, who must sue Karthik? Would Ramesh and Aditya both succeed?
A) Ramesh cannot sue Karthik, but Aditya can.

CLATGyan | Batch of 2015, 2018 & 2019 – NALSAR University of Law Page 32
B) Aditya cannot sue Karthik, but Ramesh can.
C) Karthik cannot be sued by either party, as this is a crime and not a tort.
D) Karthik can be sued by Ramesh for breach of contract, and by Aditya for breach of
his duty of care while operating upon him.

103. Principle 1 - In the case of tortious liability, a duty of care is owed to the world
at large, and anybody affected by such breach of duty of care can sue for damages.
Principle 2 - When two parties enter into a contract and there is a breach of
contractual terms, only the party to the contract affected by such breach may sue.
Facts – Sushma and her friends were having a party to celebrate their good grades in
the recent semester examinations. Sushma ordered several pizzas from a famous pizza
chain called Pizzalicious. Reveling in the party atmosphere, Ravali, Sushma‘s friend
helped herself to a slice of pizza and took a bite. After she had eaten it, she saw
something resembling part of a worm in the remaining slice of the pizza. Ravali was
shocked and greatly traumatized by this, and also fell sick subsequently. When she
recovered, she wished to sue Pizzalicious for the bad quality of pizza provided by them.
Pizzalicious, however, took the defence that they may only be sued by Sushma, who
had placed the order for the pizzas and paid for them. Decide.
A) Only Sushma can sue, as she had ordered the pizzas and the contract was between
Pizzalicious and Sushma.
B) Ravali can sue, as she sustained injury due to the worm in the pizza.
C) Sushma can sue for contractual liability, as the pizzas provided were not fit for
consumption, and Ravali can sue for tortious liability as Pizzalicious owed a general
duty of care to provide food fit for consumption.
D) None of the above.

104. Principle – Tortious liability may arise even though there is no contract between two
parties, if one of them has been unjustly enriched at the cost of the other.
Facts – Rohit hears a knock at the door, and sees that someone has come to deliver a
bag of grocery supplies. Rohit had not ordered for them, but he silently accepts the
supplies without disclosing that he had not ordered them. His neighbor, David, who
had actually ordered the supplies and paid for them is perplexed that they have not
been delivered yet.
A) Rohit is liable to either compensate David for the supplies received, or return the
goods to him, as he has obtained unjust enrichment.

CLATGyan | Batch of 2015, 2018 & 2019 – NALSAR University of Law Page 33
B) Rohit is not liable to compensate David as the goods had been delivered to him,
and it was not his fault.
C) There is no contract between David and Rohit, so David cannot recover the goods
or claim compensation for his loss. He must, instead, sue the man who wrongly
delivered the goods.
D) Both (b) and (c).

105. Principle - Violation of a legal right, with or without actual damage, gives rise
to a tort. However, actual damage without violation of a legal right does not give rise to
tortious liability.
Facts – Arpita was travelling to Ladakh for a business visit, while she was stopped by
some police officers at a check post on the highway. They detained her on the pretext
of her possessing illegal substances on her person, and restricted her from contacting
anybody who might help her in the predicament. As a result of being detained, she was
unable to fulfil the purpose of her visit, and the business deal was lost, causing losses
to her company. When she was finally released, she wished to sue the police
authorities for infringement on her fundamental rights to movement, speech and
expression. Will she succeed in these claims?
A) Arpita will succeed only in the claim for violation of her right to movement, as the
loss she suffered was due to this.
B) Arpita cannot sue for violation of her right to freedom of speech and expression, as
she was eventually released and did not suffer any real harm.
C) Both (a) and (b)
D) Arpita will succeed in both claims as there has been a violation of her legal rights in
both circumstances, and the police authorities will be liable.

106. Principle 1 - Violation of a legal right, with or without actual damage, gives rise to a
tort. However, actual damage without violation of a legal right does not give rise to
tortious liability.
Principle 2 – When a person owes a duty of care to another, and causes breach of
such duty of care, he can be said to have violated the other person‘s legal right.
Facts – Stuti studies in a law school, and she wishes to avail the University‘s
exchange programme through which she will get an opportunity to study at a foreign
law school for one semester. According to the rules to apply for the exchange
programme, she must deposit a certain sum with the university before a specified

CLATGyan | Batch of 2015, 2018 & 2019 – NALSAR University of Law Page 34
date. So, Stuti goes to the bank, which has a branch in her university‘s campus, and
hands over a cheque to be encashed. The banker, who did have the sufficient amount
of cash required, and deposited in her account, refused to encash it without giving
her any valid reason.
Stuti then goes to a different bank, gets her cheque encashed, and successfully
applies for the exchange programme. Will the banker still be liable for his act?

A) The banker will be liable, as he has refused to encash Stuti‘s cheque without a valid
reason, and has violated her legal right. Although there was no real damage that
Stuti suffered, he will still be liable.
B) No, the banker will not be liable as Stuti still managed to get the required sum from
a different bank, and successfully applied for the exchange programme. She has not
suffered any damage.
C) The banker‘s refusal to encash a cheque does not amount to violation of a legal
right, so he will not be held liable.
D) Can‘t say. The facts are inadequate to decide.

107. Principle 1 – If an act is done intentionally, the person committing the act would be
held liable irrespective of the motive behind the act.
Principle 2 - Where it can reasonably be foreseen that an act would cause harm to
the person or property of another, the person committing such act will be held liable
for acting negligently.
Principle 3 – An act committed without intention, accidentally or involuntarily,
does not give rise to liability.
Facts – Simon was greatly interested in photography, and decided to take a walk in
the woods, so he may engage in some wildlife photography. Seeing a butterfly on a
bunch of flowers, he kneels behind a bush to get a close shot of it. In the meantime,
Adams, who was in the woods to practice shooting, pulls his trigger to shoot a bird.
However, he misses his aim, and the dart ricochets off the tree and hits Simon,
causing injury to him. In the shock of the moment, Simon lets his expensive camera
drop, and the camera is broken. Simon wishes to sue Adams for injury to his person,
and for the damage to the camera.
A) Simon will succeed, as Adam‘s act was negligent and he should have foreseen that
the shooting dart would hit someone else.

CLATGyan | Batch of 2015, 2018 & 2019 – NALSAR University of Law Page 35
B) Simon will succeed, as Adam must have acted intentionally to sabotage his
photography.
C) Adams did not intend to shoot at Simon, but the act was accidental and
unforeseeable. So, he would not be held liable for the injury sustained by Simon
or for the breakage of the camera.
D) None of the above.

108. Principle - Violation of a legal right, with or without actual damage, gives rise to a
tort. However, actual damage without violation of a legal right does not give rise to
tortious liability.
Facts – Mr. Akhil lives in a locality that lies just off the main road, which supports
heavy traffic and is one of the most important roads in his city. The street on which
his house is located is, however, quiet and peaceful, and Akhil is pleased to live there.
One day, some road repair works are undertaken on the main road, and as a result,
the municipal authorities decide to divert the traffic through the street on which
Akhil resides. Akhil is greatly disturbed and annoyed by the constant sound of the
vehicles, honking, and traffic jams along the narrow street. He wishes to sue the
municipal authorities for the nuisance caused by this diversion of traffic. Will he
succeed?
A) Yes, Akhil will succeed, as his legal right to a peaceful environment is violated by
the constant noise of the vehicles.
B) Yes, Akhil will succeed, as his mental peace has been affected by the act of the
municipal authorities.
C) No, Akhil has no legal right to enjoy a peaceful street, so there has been no
violation of a legal right, despite actual damage. So, he cannot succeed in a
tortious claim.
D) Facts are inadequate to decide.

109. Principle - Violation of a legal right, with or without actual damage, gives rise to a
tort. However, actual damage without violation of a legal right does not give rise to
tortious liability.
Facts – Ayush has a shop that offers printing, Xerox, and book binding services. In
the shop, he also sells notebooks, stationery, printing inks, paper and other such
items. He has a big board outside his shop displaying all the things he deals in, and
this board is instrumental in attracting customers, who would otherwise be unaware

CLATGyan | Batch of 2015, 2018 & 2019 – NALSAR University of Law Page 36
that he sold such things. His shop is situated in a narrow street with several other
stores. Rajesh owns a book store next to him, and he installs a display shelf with all
the titles he sells, outside his shop in order to woo customers. This shelf obscures the
view to Ayush‘s board, and there is a decrease in the number of customers who buy
from him. Ayush decides to sue Rajesh and claim damages.
A) Ayush can get damages, as he has suffered damage. The display shelf of Rajesh
obscured the view of Ayush‘s board, and Rajesh had no right to do that.
B) Ayush can claim damages, but it is up to the court to grant them, as the loss
suffered is not so substantial. The shelf did not entirely block the view to the
board.
C) Ayush cannot claim damages in this case, as his legal rights have not been
violated by Rajesh in setting up a display shelf.
D) None of the above.

110. Principle – Ubi jus ibi remedium. Where there is a legal right, there is a legal
remedy.

Facts – Deepti stays at a hotel in Indonesia for three months. After about a
month, she gets fed up of the food served at the hotel, so she goes out to buy some
ingredients, vegetables, etc. and asks the hotel chef to prepare a vegetarian meal
and serve it in her room. The chef does so, but when the meal is served, she is
asked to pay for the meal. Deepti, who had already spent a lot on the ingredients,
refuses to pay for the simple vegetarian meal. The next day, at the breakfast table,
the chef refuses to serve her any food. Deepti decides to sue the chef and claim
damages.

A) Deepti will be entitled to damages as her right has been violated. She has
suffered actual as well as legal damage.
B) Although there has been actual damage, there is no legal damage; hence Deepti
will not be entitled to receive damages.
C) Can‘t say whether or not there has been legal damage.
D) Deepti will be entitled to receive damages, as the chef has unreasonably denied
serving food to her.

CLATGyan | Batch of 2015, 2018 & 2019 – NALSAR University of Law Page 37
111. Principle - When an unlawful act is committed, it is called
Malfeasance. The unlawful acts of Malfeasance are those which are
generally actionable per se and do not require proof of damage or
negligence or malice.

Facts – The estate of Mr. D‘Souza has a private orchard which is abundant in
orange trees, beautiful flowering plants, etc. Ritu, greatly awed, wishes to take a
walk in this orchard and admire its beauty. She requests the guard several times to
let her inside, so she may take a walk, but the guard refuses to do so. He informs
her that it is private property, and she can enter it only with the express
permission of Mr. D‘Souza. Fed up, Ritu quietly climbs the low fence and enters
the orchard. She plucks several exotic flowers and fruits, but is discovered by Mr.
D‘Souza, who is livid, and intends to sue her.

A) Ritu will not be liable, she has committed no wrong.

B) Ritu will not be liable as her motives were innocent, and she merely wanted to
admire the beauty of the orchard.

C) Ritu will be liable, as her act of entering private property without permission is
wrongful, irrespective of her motive to do so.

D) None of the above.

112. Principle – Whoever stores a substance which would cause damage on


escape shall be strictly liable (i.e., liable even when he has exercised
necessary care) for any damage caused by the escape of that substance.

Facts – Cobalamine Co. is a manufacturer of hydrogen peroxide which has several


industrial uses, but is also dangerous. The Company has one of its factories set up
in Dimapur, a small district. While transporting the hydrogen peroxide from that
factory to another place in huge containers, the engine of the truck carrying the
chemical got overheated, and this triggered an explosion of the concentrated
hydrogen peroxide. The truck driver, and several other people on the road were
grievously injured. The reaction was shortly contained by an expert team, and
investigations revealed that reasonable precautions had been taken to prevent such
a mishap. Will Cobalamine be liable to pay damages to those injured?

A) No, Cobalamine would not be liable, as they had taken reasonable precautions
to prevent the mishap.

CLATGyan | Batch of 2015, 2018 & 2019 – NALSAR University of Law Page 38
B) The factory authorities at Dimapur would be liable, not Cobalamine.

C) Cobalamine would be liable as hydrogen peroxide is a dangerous substance and


its escape fixes liability, despite the taking of reasonable precautions.

D) Both (b) and (c).

113. Principle – The master is liable for the wrongful acts of the servant done
in the course of employment.

Facts – Pradyuman is a mechanic working in the car mechanics shop owned by


Abhjit. Pallavi wants her Audi A8 to be serviced and cleaned, as she is planning to go
on a long drive soon. She drops off the car at the mechanic‘s, and asks Pradyuman to
have it ready in two days. Pradyuman, while trying to drive the car to the jet-cleaning
area, accidentally bumps it against the wall and a dent is created on the car door.
When Pallavi comes to take her serviced car, she is livid at the condition in which it is
returned to her, and wishes to sue for damages.

A) Only Pradyuman will be liable as he has caused the damage to the car.

B) Both Pradyuman and Abhijit will be jointly liable.

C) Abhijit will be liable to pay damages, as Pradhyuman is his servant and the damage
was caused in the course of employment.

D) Pallavi will not be entitled to claim damages as she should not have given her car to
a small mechanic such as Abhijit.

114. Principle 1 – Only a voluntary act gives rise to liability. An involuntary act
does not give rise to liability.

Principle 2 - If a particular harm can be reasonably foreseen, the person


causing such harm will be held liable.

Facts – Mr. Dinshaw, an old and grumpy man, hates stray dogs. He is on a walk with
his son, when he sees one on the street, and picks up a stone to throw at it. Perturbed,
his son grabs his hand to prevent him from doing so, but the stone was already
launched. It hits a glass window pane and breaks it. Also, while his son makes a grab
for Dinshaw‘s hand, he accidently hits a woman standing nearby, and she suffers a
nosebleed. Decide whether a claim for damages will be successful by the woman, and
by the owner of the house whose window was broken.

CLATGyan | Batch of 2015, 2018 & 2019 – NALSAR University of Law Page 39
A) The owner of the house can claim damages. It could have been foreseen that
throwing a stone on a street can cause damage.

B) The woman would also be entitled to claim damages as she suffered damage,
despite the son having committed an involuntary act.

C) The owner of the house can claim damages as it could have been foreseen that
throwing a stone on a street can cause damage. The woman would not be entitled to
damages, as the son‘s act was involuntary, and he did not intend to hit her.

D) None of the above.

115. Principle – The mere presence of malicious intent does not make a
person liable, if his act was in the exercise of a legal right.

Facts - Alexander, a competitive businessman, could not stand the fact that David‘s
bakery business was doing so well. In a bid to topple him, Alexander sets up a bakery
next to that of David, and offers cakes, bread and cookies at a much cheaper rate. He
also offers free goodies and bakes cakes in interesting flavours, which attracts all the
customers to his own bakery. As a result, David incurs huge losses and is forced to
close down his store after a few months. He is aware of Alexander‘s ploy to topple his
business, and wishes to sue him and claim damages.

A) Alexander will not be liable to pay, as he has not violated any rights of David. He
has merely exercised his own right, and his malicious intent is irrelevant.

B) Alexander acted maliciously, and will be held liable.

C) David cannot claim damages, as it is his own fault that he cannot keep up with
business competition.

D) Both (a) and (c).

116. Principle – Volenti non fit injuria - No remedy can be claimed for harm caused
through voluntary consent.

Facts – On a hot, sunny day, Avtar Singh, a traffic policeman wishes to take a break
from his work on the Jangpura-Jayanagar junction. He sees a grocery store nearby,
and decides to buy some refreshments. To get to the grocery store, he must cross the
busy road first. While doing so, he sees that a child has suddenly jumped down from
the pavement on the other side, and is attempting to cross the road, despite vehicles
hurtling past at a high speed. On seeing an approaching car that the child does not

CLATGyan | Batch of 2015, 2018 & 2019 – NALSAR University of Law Page 40
seem to be aware of, Avtar lunges forward and pushes the child out of the way.
However, the driver of the car, Mr. Takwani, is unable to stop the car in time and hits
Avtar, who sustains serious injuries, including a torn ligament. This means that Avtar
will not be able to perform his duties as a traffic policeman for at least six months. He
wishes to sue Takwani and claim damages for his loss in income, but Takwani asserts
that there was a green light, and he had committed no fault in driving at a reasonable
speed. It was Avtar who had suddenly lunged forward, giving him no time to apply the
brakes! Decide.

A) Avtar voluntarily lunged forward to save the child from being hit by the
approaching car, fully aware that there were vehicles approaching at high speeds.
Therefore, he cannot claim damages.

B) Avtar had no option but to put himself in harm‘s way, and save the innocent child.
Takwani should have been more careful while driving, and hence, he must pay
damages for the harm suffered by Avtar.

C) Avtar was being unnecessarily noble, his act was entirely voluntary, so he cannot
claim any damages.

D) Takwani was at fault, and must be prosecuted for rash and reckless driving.

Questions 17- 20 are based on a common set of Principles and Facts. Answer
accordingly.

Principle 1 – An action against violation of Fundamental Rights can only be brought


against legislative or administrative actions of the state, and not against private
actions.

Principle 2- ‗State‘ includes the Government of India, the Parliament, the state
governments and legislatures, and all local and other authorities under the control of
the government of India.

Principle 3 – The fundamental right to equality entails that equals be treated equally.

Principle 4 – No citizen shall on the grounds of caste (among other grounds), be


ineligible for an office under the State.

Principle 5 – Any law or action of the State that contravenes fundamental rights will
be void to the extent of that contravention.

CLATGyan | Batch of 2015, 2018 & 2019 – NALSAR University of Law Page 41
Facts – Nirmala and Sitara are both civil servants who have served in different
capacities for about twelve years now. Despite several transfers and changes in
designation, they have continued to remain friends. One day, they both attend a
stirring talk by a Dalit rights activist, and are so influenced by this talk, that they want
to contribute to the Dalit movement in their capacity as civil servants. They decide to
apply for managerial posts at the Department for Minority Affairs. Nirmala‘s
application for the post was rejected on the grounds that she was a Brahmin herself,
and not eligible to manage the affairs of SCs and STs. Sitara‘s application for the post
was rejected due to past animosity with Mr. Kapur, head of the Minority Affairs
Department. Mr. Kapur, in a scathing email, informed her that she would not be
considered for any post that required his approval. Nirmala and Sitara are both
enraged by these rejections, and allege that their fundamental rights have been
violated.

117. Can there be an action for violation of fundamental rights against the Department for
Minority Affairs?

A) No, there can be no action, as the Minority Affairs department is not the
Government.

B) Maybe, as Nirmala and Sitara were civil servants and employees of the State.

C) Yes, an action can lie, as the Minority Affairs Department is an authority under the
control of the Government.

D) None of the above.

118. Have the fundamental rights of Nirmala been violated in this instance?

A) Yes, as she was denied the post based on the fact that she was Brahmin and
therefore ineligible.

B) No, as the post required an SC/ ST candidate, and the law does not prevent
unequals from being treated unequally.

C) The facts as to the reason for discrimination are inadequate.

D) Yes, the very requirement of an SC/ ST candidate for the post was discriminatory.

CLATGyan | Batch of 2015, 2018 & 2019 – NALSAR University of Law Page 42
119. Have the fundamental rights of Sitara been violated?

A) No, Sitara has not been discriminated against on the basis of caste. Her
fundamental rights are hence, not violated.

B) Yes, her fundamental rights have been violated, as her application was rejected due
to the personal animosity that Mr. Kapur had. She was not treated equally with her
co-applicants.

C) No, there was no violation of fundamental rights, as the violation was not
committed by the State.

D) No, there was no violation of fundamental rights. She can sort out her animosity
with Mr. Kapur, the matter is merely personal.

120. Suppose that the Government passes a law that mandates that only persons belonging
to the SC/ ST communities can head those departments, would this law be valid?

A) No, this law would not be valid. It is overly restrictive.

B) Yes, this law would be valid. It merely lays down a requirement of eligibility.

C) No, this law would not be valid, as it discriminates on the basis of caste.

D) Can‘t say.

Questions 121-124 are based on a common set of Principles and Facts. Answer
accordingly.

Principle 1 – An offer is made when one person signifies to another, his willingness to do or
not do something, with a view to obtaining that person‘s assent.

Principle 2 – When the person to whom the offer is made signifies his assent thereto, the
offer is said to be accepted.

Principle 3 - When, at the desire of the promisor, the promisee or any other person has
done or abstained from doing, or does or abstains from doing, or promises to do or to abstain
from doing something, such act or abstinence or promise is called a consideration for the
promise.

Principle 4 – An agreement without consideration is not enforceable under law.

CLATGyan | Batch of 2015, 2018 & 2019 – NALSAR University of Law Page 43
Principle 5 – Parties to a contract must agree upon the same thing in the same sense.

Principle 6 – Parties competent to contract must be major, and of sound mind.

Facts – Sonia is a gardener in the mansion of Mr. Kapoor. Mr. Kapoor is extremely fond of
his pet dog, Cooper. One day, Kapoor realizes that Cooper is missing from his kennel. He is
frantic with worry, and sends Sonia on a quest for the dog. Sonia sets out, pastes notices in
public places and enquires about the dog. She has been instructed to return only if she finds
the dog. In the meantime, Kapoor announces in the popular newspaper, The Statesman, that
Cooper is missing, and that anybody who finds the Labrador and brings him back would be
awarded a sum of Rs. 10,000.

The next day, Kaza brings a brown Labrador with him and claims the reward. Mr. Kapoor
immediately realizes that the dog is not Cooper, and refuses to pay the sum. Kaza asserts that
the dog is named Cooper, is a brown Labrador resembling the picture in the newspaper, and
since he has accepted the offer by bringing the dog, there is a valid contract formed, and
Kapoor is bound to pay the consideration of Rs. 10,000.

Three days later, Sonia manages to find Cooper in a nearby park, and brings him back home.
Kapoor is overjoyed, and raises Sonia‘s salary from Rs. 12,000 per month to Rs. 20,000.
Sonia then discovers the notice in the newspaper, goes to Kapoor, and claims the reward.
Kapoor refuses to pay, as he as already given her an increment in salary.

121. Does Kaza‘s argument stand? Has there been a valid contract formed between Kaza
and Kapoor?

A) Yes. Kapoor has made an offer through the notice in the paper. By bringing back
the dog, Kaza has impliedly accepted the offer. There is also a valid consideration
promised, and Kapoor is now bound to pay the same.

B) No, Kaza has committed fraud, therefore, the contract stands vitiated.

C) The facts are unclear.

D) No, there is no contract, as the parties have not agreed upon the same thing in the
same sense. Kaza‘s acceptance relates to a brown Labrador. Kapoor‘s offer relates
to Cooper, his brown Labrador.

122. Is Sonia liable to claim the reward? Would Kapoor have to pay her the amount?

CLATGyan | Batch of 2015, 2018 & 2019 – NALSAR University of Law Page 44
A) Yes, Sonia can claim the reward, as she has impliedly accepted the offer by bringing
Cooper back.

B) Yes, Sonia can claim the reward and Kapoor must pay her, as a valid contract has
been formed. Both Sonia and Kapoor agreed upon the same thing in the same
sense.

C) No, Sonia cannot claim the reward, as she did not know of the offer in the first
place. An offer that she does not know of, hence, cannot be accepted. Hence, there
has been no acceptance on her part, and without acceptance, a contract cannot be
formed.

D) None of the above.

123. Suppose that Harsha, Kapoor‘s neighbour, while taking a walk in the park, recognizes
Cooper and brings him back. He is unaware of the newspaper notice. Can he claim the
reward?

A) No, he cannot claim the reward, as there has been no knowledge of the offer, and
hence no acceptance.

B) Yes, he can claim the reward, as he could have been expected to know of the offer,
being Kapoor‘s neighbour.

C) No, he cannot claim the reward, as there is no contract.

D) He can claim the reward, as the offer was a public one, and it must be assumed that
the offer is made to the world at large. It does not matter if every person has
specific knowledge of the offer. It suffices that the offer was made in a newspaper.

124. Suppose that Pix and Twix, two eight-year-old children who live in the nearby slum,
have had Cooper with them for three days. They then see the newspaper notice, and
take Cooper back to Kapoor. Can they claim the reward?

a) Yes, they can claim the reward, as a valid contract has been formed when they
impliedly accepted the offer by bringing Cooper back.

b) Yes, they can claim the reward as there has been acceptance of the offer and
agreement upon the same thing in the same sense.

CLATGyan | Batch of 2015, 2018 & 2019 – NALSAR University of Law Page 45
C) No, they cannot claim the reward, as they have not satisfied the terms of the offer.
They already had Cooper with them, and they did not go to the effort of finding
Cooper and then bringing him back.

D) No, they cannot claim the reward, as they are minors. Hence, no contract has been
formed and Kapoor is not liable to pay the sum.

125. Principle – Volenti non fit injuria – One who voluntarily consents to harm, cannot
seek damages.

Facts – Jitesh is a tennis player with the NALSAR Tennis Association. One evening,
while playing an intense match of tennis, the Vice Chancellor of the university decides
to visit the tennis court to watch the match. He enters the court, seats himself at the
stands and watches the progress of the match. Accidentally, he drops his watch on the
court, and Jitesh steps on it while trying to hit a backhand. The watch is a Rolex, and
has been shattered to pieces. Jitesh is terrified of disciplinary action that might be
taken against him, and is also upset that he has broken a Rolex, endorsed by his
favourite player, Federer. He approaches you for advice.

A) Jitesh will not be held liable, as the principle of volenti non fit injuria would apply. The
Vice Chancellor can be reasonably expected to foresee that a watch dropped on the
tennis court could be stepped on during a match.

B) Jitesh will be liable. He has stepped on the Vice Chancellor‘s watch, when he could
have avoided it. He must pay damages.

C) If Jitesh had caused injury or harm by hitting the tennis ball during the match, that
can be reasonably expected. However, he stepped on it. This cannot be reasonably
expected, and hence, he must be held liable.

D) None of the above.

Questions 126-128 are based on a common set of principles and facts. Answer
accordingly.

Principle 1 – An offer may be made to the world at large, but a contract can only be
made with the person who performs the conditions of the offer and thereby accepts the
offer.

CLATGyan | Batch of 2015, 2018 & 2019 – NALSAR University of Law Page 46
Principle 2 – In order for a contract to be valid, there must be an intention to enter
into legal relations.

Facts – Fight Anopheles Inc. is a company that manufactures mosquito repellant


coils, sprays, etc. They have invented a new mosquito repellant cream that insulates
specifically from the lethal bite of the Anopheles mosquito, the transmitting agent for
malaria. In order to advertise and increase sales of this repellant cream, they put out
an advertisement in the popular daily, The Logical Citizen. It claims that Fight
Anopheles Inc. shall pay a sum of Rs. 5000 to anyone who contracts malaria despite
having used the repellant cream. Further, it states that Rs. 50,000 has already been
deposited at the Enigma Bank as a mark of Anopheles‘ sincerity in the matter. Ms.
Dunbar comes across this advertisement, and is intrigued by the product. Hoping that
it will keep the mosquito menace at bay, she purchases it and uses it every day, but
contracts malaria nonetheless. Upset that she had put her faith in an absolutely
ineffective product, she claims that the sum of Rs. 5000 be paid to her immediately.

126. Has there been an intention to contract?

A) No, there has been no intention to contract. The company merely wanted to
increase sales and it is after all, an advertisement.

B) No, there is no intention to contract; the offer seems to be a mere puffing


advertisement to draw attention to the product. The promise of paying an amount
only serves to indicate the efficacy of the product.

C) Yes, there has been an intention to contract. The advertisement makes an offer to
the world at large, and the intention to enter into legal relations is also indicated by
the deposit of Rs. 50,000 in the bank.

D) None of the above.

127. Is Anopheles liable to pay?

A) Yes, Anopheles is liable to pay, as a valid contract gets formed when Ms. Dunbar
performs the conditions of the offer and thereby accepts it.

B) Yes, Anopheles is liable to pay, as there has been an intention to contract.

C) Yes, Anopheles is liable to pay, as they have misled customers through the
advertisement.

CLATGyan | Batch of 2015, 2018 & 2019 – NALSAR University of Law Page 47
D) No, Anopheles would not be liable to pay. The advertisement is a mere puff, there is
no intention to enter into legal relations.

128. Ms. Sharp, who also saw the advertisement, decides to test the repellant cream for its
efficacy. Despite its usage, she contracts malaria, and wishes to sue Anopheles for
misrepresentation and claim the sum. Suppose that Anopheles has already paid Ms.
Dunbar the sum, would they still be liable to pay Ms. Sharp?

A) Anopheles will be liable to pay, as the offer is not made to a specific person.
Anybody who fulfills the conditions can accept the offer, and a contract is thereby
formed, which makes Anopheles liable.

B) No, Anopheles need not pay, as they have already paid Ms. Dunbar and the contract
has been concluded.

C) Yes, Anopheles will be liable to pay, as they made a promise in the advertisement
and are bound to keep up their word.

D) Can‘t say. Depends on the specific facts of Ms. Sharp‘s usage of the repellant cream.

Questions 129-133 are based on a common set of principles and facts. Answer
accordingly.

Principle 1 – An offer is the final expression of willingness by the offeror to be bound


by his offer, should the other party choose to accept it.

Principle 2 – An invitation to offer merely invites others to make offers that one may
or may not accept. It is not an offer in itself.

Principle 3 – Acceptance needs to be communicated to the offeror.

Facts – Lockhart writes to his closest friend and aide, Helga, ―I wish to sell my book, A
Cauldron Full of Hot Strong Love, for 200 Galleons.‖

He then sends a letter to Sirius – ―Sirius, I have been meaning to sell my book, the one
that chronicles our adolescent experiments in love and other adventures. What do you
think?‖

He also writes to Harry – ―Harry, I am your biggest fan. Would you like to buy my
book, A Cauldron Full of Hot Strong Love, for 200 Galleons?‖ and to Ron - ―Ron, I see
that you are in need of advice on matters of the heart. Why don‘t you buy my book, A

CLATGyan | Batch of 2015, 2018 & 2019 – NALSAR University of Law Page 48
Cauldron Full of Hot Strong Love, for 200 Galleons? Willing to offer a discount of
20%.‖

Harry sees the letter, makes a mental note of Lockhart‘s message, and throws the letter
in the dustbin. He intends to buy the book, and makes a reminder note to write back to
Lockhart and send him a sum of 200 Galleons.

Ron receives the letter and immediately writes back to Lockhart saying that he would
like to buy the book. He posts the letter, the letter reaches Lockhart, but he then
decides not to offer the discount or even sell the book to Ron.

129. Has Lockhart made an offer to Helga?

A) Yes, he has made an offer, as he says that he wishes to sell the book for a specified
amount.

B) Yes, he has made an offer to Helga by writing to her that he wants to sell the book.

C) This is not an offer, it‘s just a letter sent to Helga.

D) No, Lockhart has not made an offer. He has merely informed Helga of his desire to
sell.

130. Has Lockhart made Sirius an offer?

A) Yes, by informing Sirius of the desire to sell, he makes an offer to sell the book.

B) No, this is not an offer. He has merely asked Sirius what he thinks of Lockhart‘s
desire to sell the book.

C) Can‘t say if this is an offer or invitation to offer.

D) None of the above.

131. Sirius, on receiving the letter, writes back to Lockhart and says that he would like to
buy the book from him, and asks him to quote a price. Can this be considered as
acceptance, and has a contract been formed between Lockhart and Sirius?

A) Yes, Sirius has communicated acceptance here. Therefore, a contract has been
formed.

B) Can‘t really say if this is an offer or an acceptance by Sirius.

CLATGyan | Batch of 2015, 2018 & 2019 – NALSAR University of Law Page 49
C) No, Sirius has actually made an offer to buy Lockhart‘s book from him. It is
Lockhart who has to accept that offer.

D) No, this would not be considered as acceptance, as Sirius is merely asking him to
quote a price. Lockhart has made no offer in the first place, that Sirius has the
option of accepting.

132. Is there a contract between Harry and Lockhart, considering that Harry intends to
buy the book?

A) No, there is no contract, as Harry has not communicated his acceptance to


Lockhart.

B) No, there is no contract as Lockhart has not made Harry an offer.

C) Yes, there is a contract. Harry intends to buy the book, and this can be seen as
implicit acceptance.

D) None of the above.

133. Has Lockhart made Ron an offer? Is there a contract formed through Ron‘s
acceptance?

A) Yes, he specifies what he wants to sell, and at what price. Therefore, it is an offer.
Yes, there is a contract formed.

B) Yes, the letter to Ron can be considered an offer and a contract is formed when Ron
accepts the offer.

C) No, it is not an offer, but rather an invitation to offer. If Ron expresses interest in
buying the book, it is at the option of Lockhart to sell and to offer the discount.
Therefore, when Ron writes back, this is not an acceptance and no contract is
formed.

D) None of the above.

Questions 134-136 are based on a common set of principles and facts. Answer accordingly.

CLATGyan | Batch of 2015, 2018 & 2019 – NALSAR University of Law Page 50
Principle 1: Any person (Principal) authorizing another person (Agent) to do a certain act
will be liable for all acts of such person done within the course of employment. The tests of
control and direction must be complied with.
Principle 2: A wrongful act authorized by the Principal as well as a lawful act done in a
wrongful manner would be considered to have been within the course of employment unless
specific directions were given regarding the mode of performance of the act.
Principle 3: For an act to fall outside the scope of employment, the act should either have
been performed after the authorized act had come to an end or must be of such nature that it
can be completely divorced from the authorized act.
Principle 4: Such a relationship need not be a long term arrangement and can be set up for
one specific transaction.
Facts: Aggubai instructed her long standing childhood friend Annubai to go to Palampur and
strike a deal with Tagesh, a spirit supplier, for the purchase of 1000 bottles of McDowell‘s
No.1 whisky, which Aggubai intended to sell at her retail store in Mumbai. Annubai was also
instructed to keep in touch with Aggubai over phone regarding the deal. Accordingly,
Annubai took a train to Palampur, planned a meeting with Tagesh and made the requisite
purchase.

134. Did Annubai and Aggubai have a principal agent relationship?


A) No. Even though Annubai was acting on the instructions of Aggubai she was
beyond her control once she left for Palampur.
B) No. They were childhood friends and Annubai was only helping her friend.
C) Yes. Annubai was acting on the instructions of Aggubai and was under the control
and direction of Aggubai.
D) None of the above.

135. On her way back, Annubai decided to drive down for a part of the journey as
her friend Bhavinder told her it was a scenic drive. Annubai got tempted and downed 4
bottles of whisky in the car. Owing to her inebriated state, Annubai sped and her car
ran into a family of three sleeping on the footpath. Who will be liable for the accident?
A) Annubai. She was driving in an inebriated condition and she should alone be liable
for it.
B) Aggubai. Annubai was transporting the bottles of spirit on her instructions and the
employment would be completed only once the spirit was handed over to Aggubai
at Mumbai.

CLATGyan | Batch of 2015, 2018 & 2019 – NALSAR University of Law Page 51
C) Annubai. She was no longer operating within the course of her employment. Her
employment was over once the purchase was made from Tagesh.
D) The people sleeping on the footpath. It was a clear case of volenti non fit injuria.

136. Had Aggubai clearly instructed Annubai to travel only by train, who would then
be liable for the accident?
A) Annubai. By deciding to drive she clearly operated outside the course of her
employment.
B) Aggubai. Even though she asked Annubai to travel by train, she never asked
Annubai not to consume alcohol. The accident happened because of the alcohol
not because Annubai travelled by car.
C) Both Annubai and Aggubai.
D)None of the above.

137. Principle- Where one moves another‘s property without the owner‘s consent
and does so with a dishonest intention, he commits the offence of theft.
Facts: Ramesh finds Suresh‘s I-pod unattended in the library. He picks it up with the
intention of giving it to Pranesh who is Suresh‘s roommate to return it to Suresh.
Unable to find Pranesh, Ramesh decides to listen to music for a while to pass time
when Chintoo approaches him and says- ―Wow. What a cool music player‖. Ramesh
responds with ―Yeah, just got it last week‖. After some time, Suresh spots Ramesh
using his I-pod and accuses him of theft. Is Ramesh liable?

A) Yes. He told Chintoo the I-pod was his. He had already committed theft by then.
B) No. He had no dishonest intention as he wanted to return the I-pod to Pranesh
and was only using it for a short while.
C) Yes. He used it without Suresh‘s permission.
D) None of the above.

138. Principle- The delivery of possession is an essential requisite of a contract of


bailment. It is different from custody. After the delivery of possession, the bailee has
full control over the goods bailed and he must take care of the goods as if they were his
own.
Facts - Berry walks into Straw‘s office for a meeting and as he was entering, Shelly,
Straw‘s employee, took his coat off and hung it on a hook behind the door of the

CLATGyan | Batch of 2015, 2018 & 2019 – NALSAR University of Law Page 52
meeting room without being asked to do so. Shelly had been instructed to do so for
everyone attending the meeting. After the meeting when Berry asked for his coat, it
was missing. Is Straw liable?
A) No. Straw is not liable. Berry should have looked after his own coat.
B) Yes. Shelly was a bailee in absolute control over it and decided herself where
and how to keep Berry‘s coat.
C) No. Straw and Shelly were only trying to be courteous to whoever attended a
meeting at their office.
D) None of the above.

139. Jack was a high-profile detective. He decided to outsource some work to Jones,
an efficient but less experienced detective. Jones had been appreciated by many a client
for his high work ethics. Jack assigned specific cases to Jones. The papers related to
these cases were handed over to Jones every morning and locked in a cupboard located
in Jones room every evening after making sure no paper/s had been removed. The keys
to this cupboard were always in Jack‘s possession. One morning, when Jack went to
Jones‘ office to hand over the papers to him as usual and unlocked the cupboard,
several important papers were missing. He filed a suit against Jones. Will he succeed?
A) Yes. The papers went missing from Jones‘ office.
B) Yes. It is possible Jones slipped out important papers the previous day.
C) No. Even though the papers were physically placed in Jones‘ office he never had
control over them as Jack retained the keys. There was no delivery of
possession.
D) No. Jones was a man of high work ethics. The Court would take his personal
character into account.

140. Principle- Nothing is an offence merely by reason of it being done with the knowledge
that it is likely to cause harm if it is done without any criminal intent and in good faith
for the purpose of preventing a greater harm.

Facts- Rangeela who was driving a car met with an accident with a truck and sustained
serious injuries on her lower abdomen. Dr. Manohar Lohiya was carrying out a surgery
to repair tissue injuries sustained by Rangeela close to her uterus. In the middle of the
surgery he found a metal piece lodged dangerously on Rangeela‘s uterine wall. He

CLATGyan | Batch of 2015, 2018 & 2019 – NALSAR University of Law Page 53
made an immediate decision to pull the piece out. However, his act led to unexpected
and excessive bleeding causing Rangeela‘s death. Is Lohiya liable for murder?
A) Yes. He recklessly pulled the metal piece and that lead to Rangeela‘s death.
B) No. He had to make an immediate decision and he did so in good faith in order
to prevent greater harm to Rangeela.
C) No. It was a clear accident.
D) Yes. He will held liable for culpable homicide.

141. The 99th Constitutional Amendment Act has sacked the system of Collegium for
appointing and transferring judges of the High Courts and the Supreme Court. Who
amongst the following is not a member of the recently constituted National Judicial
Appointments Commission?
A) Second senior-most judge of the Supreme Court

B) Union Minister for Law & Justice


C) Attorney General of India
D) Chief Justice of India

142. Forty years ago, India saw emergency being put in force for about two years.
According to Article 352 of the Constitution, the President has the power to proclaim
emergency. Why then is Ms. Indira Gandhi, the then Prime Minister of India, is accused
of imposing emergency when it was a proclamation made by the then President,
Fakhruddin Ali Ahmed?

A) Because the President needs prior permission of the Prime Minister to proclaim
emergency.

B) Because the President acts according to the aid and advice of the Council of
Ministers headed by the Prime Minister.
C) Because the Prime Minister did not order the election commission to conduct the
general elections, and the President has to proclaim emergency if elections are not
conducted on time.
D) Because Ms. Indira Gandhi got the Constitution amended and took away the
President‘s power of proclaiming emergency and accorded it to the Prime Minister.

CLATGyan | Batch of 2015, 2018 & 2019 – NALSAR University of Law Page 54
143. In a recent judgement, the Supreme Court has held Section 66 (A) of the
Information Technology Act, 2000 to be unconstitutional. Which fundamental right did
this section violate?

A) Right to equality.
B) Right to freedom of speech and expression.
C) Right to privacy.
D) Right to independence of thought, belief, and faith.

144. The Goods and Services Tax Bill passed by the Lok Sabha, and pending before
the Rajya Sabha, has created extreme furore. What is the nature of the Goods and
Services Tax?

A) Indirect Tax.

B) Direct Tax.
C) Special Tax for luxurious goods.
D) Exemption of tax for foreign investments.

145. The recent land-swap agreement with Bangladesh required our Parliament to
pass an amendment to the Constitution. According to Article 368, an amendment to the
Constitution requires a special majority. What is a ‗special majority‘?

A) A ‗yes‘ from majority of the political parties in the Parliament.


B) A ‗yes‘ from majority of the present and voting members of the Parliament.
C) A ‗yes‘ from two-thirds of the present and voting members of the Parliament.
D) A ‗yes‘ from two-thirds of the total strength of the Parliament.

146. Which of the following is not a part of the term ‗Parliament‘?

A) The President
B) House of People
C) Council of States
D) The Prime Minister

147. Which of the following fields of law does the Government of the NCT of Delhi
has the power to govern?

CLATGyan | Batch of 2015, 2018 & 2019 – NALSAR University of Law Page 55
A) Law and order
B) Police
C) Land
D) Electricity

148. Which of the following organisations are set-up under the Constitution of India?

A) Planning Commission
B) NITI Aayog
C) National Development Council
D) Finance Commissio
149. Who is the Chairperson of the National Ganga River Basin Authority?

A) The Minister of Environment and Forest


B) The Minister of Water Resources
C) The Prime Minister
D) The President

150. Why is the Indian Political System called the ‗Westminister System‘?

A) Because all the Ministers sit on the west side on the Parliament
B) Because the Government in power occupied the West wing of the Rashtrapathi Bhavan
C) Because the Parliament of the United Kingdom is in a building called the Palace of
Westminister
D) Because the system of democracy that India follows has been influenced by the
countries in the western hemisphere.

CLATGyan | Batch of 2015, 2018 & 2019 – NALSAR University of Law Page 56
SECTION 5: GENERAL KNOWLEDGE

151. Consider the following statements regarding the newly established Asian
Infrastructure Investment Bank.

Statement 1: The Bank will be headquartered at Beijing, China.


Statement 2: India is the largest contributor to the Bank‘s funds.
Statement 3: The Bank is a part of the BRICS organisation
Which of the above statements are correct?
A) 1 & 2
B) 2 & 3
C) Only 1
D) Only 3

152. INS Vikrant was docked at the Cochin Shipyard recently, and is expected to be
inducted into the Indian Naval Service in a couple of years. What is so exciting about
INS Vikrant?

A) It is a stealth submarine which can survive the high pressure of the deep sea beds.
B) It is a nuclear powered submarine which can cause a mini-Tsunami to destroy the
enemy‘s coastal security.
C) It is an almost invisible transparent submarine which, a few people say, is the Harry
Potter invisibility cloak in reality.
D) It is an aircraft carrier developed entirely in India.

153. Which of the following is correctly matched?

1. Black Revolution : Production of Coal


2. White Revolution : Production of Milk
3. Round Revolution : Production of Potato
4. Silver Revolution : Production of Cotton
A) 1, 2, & 3
B) 2, 3, & 4

CLATGyan | Batch of 2015, 2018 & 2019 – NALSAR University of Law Page 57
C) 1, 2, & 4
D) All of the above
154. Sania Mirza has won the Wimbledon Women‘s Doubles Title – 2015. Who was her
partner?
A) Maria Sharapova
B) Martina Hingis
C) Leader Paes
D) Serena Williams

155. Across how many States in India does the Tropic of Capricorn pass though?
A) Eight
B) Zero
C) Six
D) Five

156. Recently, India entered into a Memorandum of Understanding with another country to
set-up the PACESetter Fund for PEACE (Promoting Energy Access through Clean
Energy). Which country?
A) Australia
B) France
C) United States of America
D) Mongolia

157. Which of the following political parties is not a part of the National Democratic
Alliance headed by the Bharatiya Janata Party?
A) Shiv Sena
B) Telugu Desam Party
C) Shiromani Akali Dal
D) Telangana Rastra Samithi

158. Waterloo is a town in Europe where the Battle of Waterloo was fought between
Napolean and the Seventh Coalition. In which country is Waterloo currently?
A) France
B) Netherlands
C) Belgium

CLATGyan | Batch of 2015, 2018 & 2019 – NALSAR University of Law Page 58
D) Germany

159. The Regional Integrated Multi-Hazard Early Warning System for Africa and
Asia (RIMES) is an international and intergovernmental institution established after
the Tsunami struck in the year 2004. The purpose is to have an early warning to take
precautions against earthquakes and other natural disasters. Where is its warning
system located?
A) India
B) Thailand
C) Sri Lanka
D) Indonesia

160. The Indian Space Research Organisation (ISRO) has its headquarter at:
A) Sriharikota
B) Chennai
C) Bangalore
D) Hyderabad

161. Kharif crops are sown during which season:


A) Monsoon
B) Winter
C) Spring
D) Summer

162. ‗Arab Spring‘ is a term that has been in the news very frequently in the last five
years. What does it denote?
A) The rise in prices of oil due to the United States‘ presence in the Arab countries
B) Various revolutions across Arab countries to dethrone the monarchy and adopt
democracy
C) The discovery of geothermal energy in the Arab countries, thereby reducing the oil
prices due to competition
D) The attack of United States‘ on various countries in the Arab region

163. You have boarded a flight from New Delhi to go to Moscow. All engines of the
aircraft fail, and you are tied to a parachute and thrown away from the plane by the air

CLATGyan | Batch of 2015, 2018 & 2019 – NALSAR University of Law Page 59
hostess. You land in the middle of a city. People there tell you that the city‘s name is
Ulan Bator. Which country are you in?
A) Russia
B) China
C) Mongolia
D) Tajikistan

164. Vinicius and Tom are the official mascots of:


A) 2016 Summer Olympics
B) 2016 Paralympics
C) 2014 FIFA World Cup
D) A & B

165. The Repo Rate is the rate of interest at which the Reserve Bank of India (RBI) lends
short term loans to the banks. The Governor of the RBI is an important functionary to
decide when to reduce or increase the Repo Rate. Who is the Governor of the RBI
currently?
A) Bimal Jalan
B) Raghuram Rajan
C) Arun Jaitley
D) Smriti Irani

166. Which of the following recently launched Government schemes deals with life
insurance?
A) Pradhan Mantri Jan Dhan Yojna
B) Pradhan Mantri Suraksha Bima Yojana
C) Pradhan Mantri Jeevan Jyoti Bima Yojna
D) Both B & C

167. Which of the following persons have not been awarded the Bharat Ratna?
A) Sachin Tendulkar
B) CNR Rao
C) Madan Mohan Malviya
D) P C Mahalanobis

CLATGyan | Batch of 2015, 2018 & 2019 – NALSAR University of Law Page 60
168. The Prime Minister is the ex-officio President of the Nehru Memorial Museum
& Library. Recently, Narendra Modi was appointed as its President by the President of
India. What is the subject matter of this library?
A) Historical records of the years when Jawaharlal Nehru was the prime minister
B) Collection of literature on foreign affairs with specific emphasis on Non-Alignment
C) Collection of books on the SAARC countries and their history
D) A collection of the history and happenings of independence movement of India

169. Which of the following States of India does not share its border with
Bangladesh?
A) Meghalaya
B) Tripura
C) Assam
D) Nagaland

170. What is ‗Pashu Poshan‘?


A) A Government scheme to help out debt ridden cattle pastoralists
B) An indicator of healthy meat developed by the Ministry of Health
C) A mobile application developed by the National Dairy Development Board
D) It is the chart made by the National Dairy Development Board which suggests the
correct amounts of foods for cattle

171. X on 27th May 2015 abolished the death penalty after a veto override was passed
through its legislature. With this ban, X has joined Washington DC and 18 other states.
In four decades, it is also the first traditionally conservative state to do so. Name X.
A) Maryland
B) Nebraska
C) Connecticut
D) Illinois

172. X has served as the 8th President of FIFA (the Federation Internationale de
Football Association) since June 1998. On 2nd June 2015, after the US government
officials indicted several current and former FIFA officials and sports marketing
companies for bribery and money laundering, X announced that he would call for
elections and that he would not stand in these elections but would remain in his

CLATGyan | Batch of 2015, 2018 & 2019 – NALSAR University of Law Page 61
position until an extraordinary FIFA Congress could be held for his successor to be
appointed. Identify X.
A) Thomas Bach
B) Prince Ali bin Hussein
C) Sepp Blatter
D) Jim Boyce

173. Forum for Human Rights and Justice organised a Marathon named as Ajanmi
Beti ke liye Daud (Protecting the Unborn Girl) in X. The objective of this Marathon is to
save the girl child and arrest the declining sex ratio in the state. X is one among the ten
worst states wherein the sex ratio is worst. It has a child sex ratio of 906 against 1000
females. X is which Indian state?
A) Himachal Pradesh
B) Haryana
C) Chhattisgarh
D) Bihar

174. Researchers have discovered the driest place on the Earth in the Atacama
Desert in Chile and is being referred to as X. The discovery was presented in the paper
―Discovery and microbial content of the driest site of the hyperarid Atacama Desert,
Chile‖, published in March 2015 in the journal Environmental Microbiology Reports. X
is as dry as Mars and has a mean atmospheric relative humidity (RH) of 17.3%. It has a
soil RH of a constant 14% at a depth of one meter. Identify X.
A) Yungay Region
B) Aswan
C) Ica
D) Maria Elena South

175. X was an American author, best known for the film adaptation of his novel
Dances with Wolves. He has also won the Academy Award for Best Writing (Adapted
Screenplay) and the Golden Globe Award for Best Screenplay for the same. He died on
May 2nd 2015. Name X.
A) Mario Puzo
B) James Toback
C) Michael Blake

CLATGyan | Batch of 2015, 2018 & 2019 – NALSAR University of Law Page 62
D) Tom Schulman

176. X is a 2015 French drama film directed by Jacques Audiard. It was partly
inspired by Montesquieu‘s Persian Letters. The film tells the story of three Tamil
refugees who flee the civil war-ravaged Sri Lanka and come to France in the hope of
reconstructing their lives. The film won the Palme d‘Or at the 2015 Cannes Film
Festival. What is X?
A) Ice and the Sky
B) Dheepan
C) Chronic
D) Rams

177. X is a series of 15 paintings and numerous drawings by Spanish cubist artist Y.


The entire series of X was bought by Victor and Sally Ganz from the Galerie Louise
Leiris in Paris for $212,500 in June 1956. The Version ‗O‘ of X was sold for $179.4
million in a Christie‘s auction in New York, the highest price ever paid for an artwork at
auction. Identify X and Y.
A) Les Femmes d‘Alger by Pablo Picasso
B) Fanny Tellier by Eugene Delacroix
C) The Weeping Woman by Pablo Picasso
D) La Femme au Cheval by Jean Metzinger

178. X is a military base located in Tajikistan. It is operated by the Indian Air Force
in collaboration with the Tajikistan Air Force. X is India‘s first and only military base
outside its territory. Name X.
A. Halwara
B. Hindon
C. Siachen
D. Farkhor

179. The Arignar Anna Zoological Park also known as the Vandalur Zoo was the first
public zoo to be set up in India in 1855. It is affiliated with the Central Zoo Authority of
India. Spread over an area of 602 hectares, the park is the largest zoological garden in
India. Where is this zoo located?
A) Madurai

CLATGyan | Batch of 2015, 2018 & 2019 – NALSAR University of Law Page 63
B) Bangalore
C) Kochi
D) Chennai

180. Hell‘s Angel is a 1994 Channel 4 television documentary about X by Christopher


Hitchens, a precursor to his book, The Missionary Position The film claims that X urged
the poor to accept their fate, while the rich are portrayed as being favoured by God. Who
is X?
A) David Livingstone
B) Hudson Taylor
C) Mother Teresa
D) Jim Elliot

181. The term for an immensely difficult task is derived from the legend of Hercules.
In the same manner, from which Greek figure is the term for an impossible task derived
from?
A) Atlas
B) Narcissus
C) Hephaestus
D) Sisyphus

182. Because of the long-term effects of inflation, notably the significant increase of
movie theatre ticket prices, the list unadjusted for inflations gives far more weight to
later films. X is generally considered to be the most successful film, with Guinness
World Records in 2014 estimating its adjusted global gross at $3.4 billion. Name X.
A) Avatar
B) Gone With The Wind
C) Titanic
D) The Sound of Music

183. X was a French economist whose reputation was established through his Melanges
economiques (1857) essays and a lecture series given at the University of Montpellier.
He was an advocate of free trade and adherent to the ideas of Richard Cobden. He was
the President of the Society of Political Economy for 70 years. In addition to his

CLATGyan | Batch of 2015, 2018 & 2019 – NALSAR University of Law Page 64
achievements, he was also the joint winner of the first Nobel Peace Prize along with
Henry Dunant in 1901. Who is X?
A) Randal Cremer
B) Charles Gobat
C) Frederic Passy
D) Tobias Asser

184. X, also known as Hind-di-Chaadar (Protector of India) for protecting Hindus


against forced conversion in the hands of Muslims under Aurangzeb, became the 9 th
Guru of Sikhs on 16th April 1664, a position earlier occupied by his grand-nephew, Guru
Har Kishan. X was publicly executed in 1675 on the orders of Mughal Emperor
Aurangzeb in Delhi for resisting the forced conversions of Hindus in Kashmir and
himself refusing to convert to Islam. Name X.
A) Guru Hardev
B) Guru Tej Bahadur
C) Guru Har Gobind
D) Guru Har Krishnan

185. Direct Action Day, (16th August 1946) also known as the Great Calcutta Killings,
was a day of widespread riot and manslaughter between Hindus and Muslims in the
city of Calcutta in the Bengal province of British India. The ‗Direct Action‘ was
announced by the X. What/Who is X?
A) Mahatma Gandhi
B) Indian National Congress
C) British Government
D) Muslim League

186. Who is the author of the collected works of the ‗Canterbury Tales‘?
A) Geoffrey Chaucer
B) E.L. James
C) Mark Twain
D) R.L. Stevenson

187. Which ruler issued a decree equating the value of bronze and copper coins with
that of silver, thus causing a near collapse of his economy?

CLATGyan | Batch of 2015, 2018 & 2019 – NALSAR University of Law Page 65
A) Firoz Shah Tughluq
B) Iltutmish
C) Aurangzeb
D) Muhammed Shah Tughluq

188. Where is the Food and Agriculture Organisation based?


A) Rome
B) Paris
C) Geneva
D) New York

189. Who was the first Field Marshal of India?


A) KM Cariappa
B) Sam Manekshaw
C) Jatinder Kumar Bajaj
D) Mihir Singh

190. Who is the author of the epic Anna Karenina?


A) Fyodor Dostoevsky
B) Dante
C) Leo Tolstoy
D) Cervante

191. Who is the author of A Song of Ice and Fire?


A) J.K. Rowling
B) J.R.R. Tolkien
C) George R. R. Martin
D) Suzanne Collins

192. What is VYAPAM?


A) Madhya Pradesh Professional Examination Board
B) Vocational and Youth Progamme
C) Tamil Nadu Vocation Youth Programme
D) None of the Above

CLATGyan | Batch of 2015, 2018 & 2019 – NALSAR University of Law Page 66
193. Who is the Leader of Opposition in the Lok Sabha?
A) Malikarjun Kharge
B) Kapil Sibal
C) Digvijay Singh
D) No Leader of Opposition

194. When was the World‘s first Yoga Day celebrated?


A) June 22
B) June 21
C) June 24
D) June 23

195. Which recent legislation was the Private Member‘s Bill passed by the Rajya
Sabha?
A) The Rights of Transgender Persons Bill, 2014
B) The Information Technology (Amendment) Bill, 2015
C) Hindu Marriage (Amendment) Bill, 2014
D) None of the Above.

196. Which city recently faced a massive strike against Uber for its Uber Pop service
by the city‘s taxi drivers?
A) London
B) Paris
C) Los Angeles
D) Rome

197. Who is hosting the FIFA World Cup 2022?


A) Qatar
B) Russia
C) USA
D) England

198. Which Judge among the following is currently a Judge at International Court of
Justice, Hague?
A) H.R. Khanna

CLATGyan | Batch of 2015, 2018 & 2019 – NALSAR University of Law Page 67
B) Krishna Iyer
C) A.K. Ganguly
D) Dalveer Bhandari

199. What is Rahul Yadav famously associated for?


A) Paytm
B) Snapdeal
C) Housing.com
D) Swiggys

200. When was the national calendar of India adopted?


A) 1956
B) 1957
C) 1958
D) 1959

CLATGyan | Batch of 2015, 2018 & 2019 – NALSAR University of Law Page 68

Вам также может понравиться